subject
Mathematics, 30.03.2021 04:10 mmk4

7. Wendy is paid $12 per hour and plans to work between 30 and 35 hours per week. Identify the independent and dependent quantity in the situation. Find reasonable domain and range values.

A. weekly pay; hours worked; $360 to $420; 30 to 35 hours

B. hours worked; weekly pay; 30 to 35 hours; $360 to $420

C. weekly pay; hours worked; 30 to 35 hours; $360 to $420

D. hours worked; weekly pay; 30 to 35 hours; $0 to $420

ansver
Answers: 1

Other questions on the subject: Mathematics

image
Mathematics, 21.06.2019 12:30, slimgemmx
In pqr the obtuse angle is 105 and the shorter sides measure 4 and 7 units rounded to the nearest tenth what is the area of triangle pqr
Answers: 2
image
Mathematics, 21.06.2019 14:30, sierraseideman1023
If given an equation of a line such as -1/2x+6 how would you create an equation of a line parallel and perpendicula to this line that goes through another point such as (4,10)
Answers: 1
image
Mathematics, 21.06.2019 16:00, tgeorge95
What is the quadric regression equation that fits these data
Answers: 1
image
Mathematics, 21.06.2019 16:20, whitneyt3218
7.(03.01 lc)which set represents the range of the function shown? {(-1, 5), (2,8), (5, 3), 13, -4)} (5 points){-1, 2, 5, 13){(5, -1), (8, 2), (3,5), (-4, 13)){-4, 3, 5, 8}{-4, -1, 2, 3, 5, 5, 8, 13}
Answers: 3
You know the right answer?
7. Wendy is paid $12 per hour and plans to work between 30 and 35 hours per week. Identify the inde...

Questions in other subjects:

Konu
Mathematics, 16.12.2019 22:31